How to expand f(x)=x^2, -pi<x<pi as fourier series

Click For Summary
To expand the function f(x) = x^2 over the interval -π < x < π as a Fourier series, the correct series includes a constant term of (π^2)/3 and a summation involving cos(nx) with coefficients that depend on n^2 in the denominator. The confusion arises from the integration process, where integrating by parts twice is necessary to derive the coefficients accurately. The integration of x^2 cos(nx) leads to terms that involve 1/n when integrating the trigonometric function, which is crucial for obtaining the correct Fourier coefficients. Additionally, the term at the beginning of the answer is indeed a0/2, addressing the issue of the n=0 case leading to infinity. Understanding these integration steps clarifies the derivation of the Fourier series for the given function.
blueyellow

Homework Statement


how to expand f(x)=x^2, -pi<x<pi as Fourier series
answer at back of the book says:
f(x)=((pi^2)/3)+4sigma(from 1 to infinity) [((-1)^n)/(n^2)]cos nx
i tried the a(n) b(n) stuff but i don't see where they get the n^2 in the denominator from

when i integrated to try to find a(n) i get n on the denominator
 
Physics news on Phys.org
It seems pretty straight forward to me. In order to get rid of that "x^2" you will need to integrate by parts- twice. And each time you will get a 1/n integrating the trig function. If you want more detail on where you went wrong, we will have to see what you did. HOW did you integrate
\int_{-\pi}^\pi x^2 cos(x) dx?
 
integral of x^2 cos ((n*pi*x)/L) dx

using partial integration
integral=first term*integral of second term -[integral of (first term differentiated*integral of second term)]
first term=x^2
second term=cos...
=integral (from -pi to pi) x^2 cos((n*pi*x)/2pi) dx
=integral (from -pi to pi) x^2 cos((n*x)/2) dx
=[(2x^2)/n sin ((nx)/2)- integral of 2x sin (nx/2)] dx
=[(2x^2)/n sin ((nx)/2)- [-4x/n cos (nx/2) + 4/n cos (nx)]dx

putting in the limits
integral=(4pi^2)/n

a(n)=1/L *integral
=2pi/n

that term at the beginning of the answer is a0/2, right?
because if n=0 is put into 2pi/n, it'll be infinity!
 
Question: A clock's minute hand has length 4 and its hour hand has length 3. What is the distance between the tips at the moment when it is increasing most rapidly?(Putnam Exam Question) Answer: Making assumption that both the hands moves at constant angular velocities, the answer is ## \sqrt{7} .## But don't you think this assumption is somewhat doubtful and wrong?

Similar threads

  • · Replies 3 ·
Replies
3
Views
2K
  • · Replies 14 ·
Replies
14
Views
2K
  • · Replies 1 ·
Replies
1
Views
2K
  • · Replies 2 ·
Replies
2
Views
1K
  • · Replies 1 ·
Replies
1
Views
2K
  • · Replies 5 ·
Replies
5
Views
2K
  • · Replies 16 ·
Replies
16
Views
2K
  • · Replies 3 ·
Replies
3
Views
2K
  • · Replies 1 ·
Replies
1
Views
1K
  • · Replies 6 ·
Replies
6
Views
2K